will wig curls come out of synethic wigs??? or do they stay there forever?
Answer asap for brainlist

Answers

Answer 1

Answer:

No ! The only thing that will take a wave or curl out of a synthetic wig is heat.

Step-by-step explanation:


Related Questions

which statement is true about finding the midpoint between two points located on a horizontal or vertical line ?

A . When finding the midpoint between two points on a vertical line, keep the y-coordinate and find the average of the x-coordinate

B . When finding the midpoint between two points on vertical line, keep x-coordinate and find the average of the y-coordinates

C . When finding the midpoint between two points on a horizontal line, the x-coordinate is always 0 and the y-coordinates are averaged

D. When finding the midpoint between two points on a horizontal line , the y-coordinate is always zero and the x-coordinates are averaged

Answers

Answer: B. When finding the midpoint between two points on a vertical line, keep the x-coordinate and find the average of the y-coordinates

Nadia says the hypotenuse of this right triangle has a length of 73 because the pythagorean theorem states that (28 45) squared = 73 squared. 3 squares are positioned to form a right triangle. the small square is labeled 28, medium square is 45, and large square is not labeled. which best describes nadia’s solution?

Answers

Both the options that Nadia gives are wrong about the length of hypotenuse.

Given length of hypotenuse is 73 and options which are :

1) Because the pythagoras theorem states that [tex]28^{2} +45^{2}[/tex] squared = 73 squared.

2) 3 squares are positioned to form a right triangle.

We have to choose the correct option which shows the calculation of length of hypotenuse.

Pythagoras theorem says that in a right angled triangle the square of hypotenuse of triangle is equal to sum of the square of base and square of perpendicular.

[tex]H^{2} =P^{2} +B^{2}[/tex]

First option says that [tex]28^{2} +45^{2} =73^{2}[/tex]

784+2025=5329

2809≠5329

so it is wrong.

Second option says that the length  of hypotenuse is the sum of three squares but we have seen that the square of the length of hypotenuse is equal to sum of two square one is the square of base and the square of perpendicular.

Hence both the options are incorrect about the length of hypotenuse.

Learn more about pythagoras theorem at https://brainly.com/question/343682

#SPJ4

Answer:

C

Step-by-step explanation:

which shows a correct step in using substitution to slove the system of equation y=2x-1 4x+y=2

Answers

The solution using correct step of substitution in the equation y=2x-1 4x+y=2 is x = 1/2 and y = 0

Simultaneous equation

y = 2x - 1

4x + y = 2

Substitute the value of y in equation (1) into (2)

4x + y = 2

4x + (2x - 1) = 2

4x + 2x - 1 = 2

6x - 1 = 2

6x = 2 + 1

6x = 3

x = 3/6

x = 1/2

substitute the value of x into equation (1)

y = 2x - 1

y = 2(1/2) - 1

= 2/2 - 1

= 1 - 1

y = 0

Learn more about simultaneous equation:

https://brainly.com/question/16863577

#SPJ1

A high school baseball player has a 0.229 batting average. in one game, he gets 5 at bat. what is the probability he will get at least 2 hits in the game? round your answer to four places.

Answers

The probability he will get at least 2 hits in the game is 0.470

Given

The batting average is 0.229

In one game he gets 5 at bat

To find the probability he will get at least 2 hits we use binomial distribution.

What is Binomial distribution?

The binomial distribution is also a Frequency distribution used for the possible no. of successful outcomes in a given no. of trials in each of which there is the same probability of success.

So we have

n = 5

p = 0.299

q = 1 - p

   = 0.701

[tex]P(X) = nC_{x} p^x q^(n-x)[/tex]

P(at least 2 hits in the game) = 1 - P(less than 2 hits)

= 1 - P(0) - P(1)

[tex]= 1 - 0.701^5 - 5C1 x 0.299 x 0.701^4[/tex]

= 1 - 0.169 - 0.361

= 0.470

Learn more on binomial distribution probability here:

https://brainly.com/question/10559687

#SPJ4

A metallurgist has one alloy containing 45% copper and another containing 69% copper. How many pounds of each alloy must be use to make 42 pounds of a third alloy containing 66% copper

Answers

5.25 pounds of one alloy containing 45% copper and 36.75 pounds of 69% copper is used to make 42 pounds of 66% copper

What is an equation?

An equation is an expression that shows the relationship between two or more variables and numbers.

Let x represent the amount of 45% copper and y represent the amount of 66% copper needed to make 42 pounds of a third alloy containing 66% copper. Hence:

x + y = 42   (1)

Also:

0.45x + 0.69y = 42(0.66)   (2)

This gives:

x = 5.25, y = 36.75

5.25 pounds of one alloy containing 45% copper and 36.75 pounds of 69% copper is used to make 42 pounds of 66% copper

Find out more on equation at: https://brainly.com/question/2972832

#SPJ1

14 ) Mark Achin sells 3,600 electric motors each year . The cost of these is $ 200 each , and demand is constant throughout the year . The cost of placing an order is $ 40 , while the holding cost is $ 20 per unit per year . There are 360 working days per year and the lead - time is 5 days . What is his economic order quantity in units ? A ) 100 B ) 200 C ) 60 D 120​

Answers

Mark Achin's economic order quantity in units is D. 120​ units.

What is economic order quantity?

The economic order quantity (EOQ) is the optimal order quantity that minimizes the total costs (ordering, receiving, and holding inventory).

The formula for the economic order quantity is the square root of [2(setup costs)(demand rate)] / holding costs.

Data and Calculations:

Annual demand = 3,600

Cost price per unit = $200

Ordering cost = $40

Holding cost per unit = $20

Working days per year = 360

Lead time = 5 days

Economic order quantity (EOQ) = square root of: [2(setup costs)(demand rate)] / holding costs.

= square root of (2 x $40 x 3,600)/$20

= 120

Thus, Mark Achin's economic order quantity in units is D. 120​ units.

Learn more about the economic order quantity at https://brainly.com/question/13386271

#SPJ1

A local hospital currently has 184 male patients. If the ratio of male to female patients is 4:3, how many female patients are there in the hospital?

Answers

There are 138 female patients in the hospital.

Solving word problems involving ratios.

The solution to the word problem involving ratios in the given problem can be determined as follows.

From the given information:

The total number of people in the hospital = male + female

= 184 + x(unknown)

If the ratio of male to female is 4: 3. The total ratio is 4+3 = 7

The ratio of males in the hospital is:

[tex]\mathbf{184= \dfrac{4}{7} \times x }[/tex]

[tex]\mathbf{184\times 7= 4 x }[/tex]

x = 322

The number of female patients in the hospital can now be computed as:

[tex]\mathbf{=\dfrac{3}{7}\times 322}[/tex]

= 138 female patients

Check:

The total number of people in the hospital is:

= 184 + 138

= 322 patients

Learn more about word problems here:

https://brainly.com/question/13818690

#SPJ1

M=−8r
2
+11r−6
T=−7r
2
−9r+14


M
+
T
=
M+T=M, plus, T, equals


Your answer should be a polynomial in standard form.

Answers

M + T in standard form polynomial is -15r² + 2r + 8

How to represent polynomial in standard form?

Polynomial in standard is when all its terms are arranged by decreasing order of degree.

Therefore,

M = - 8r²+ 11r - 6

T = -7r² - 9r + 14

M + T =  - 8r²+ 11r - 6 + (-7r² - 9r + 14)

M + T =  - 8r²+ 11r - 6 - 7r² - 9r + 14

combine like terms

M + T = - 8r² - 7r² + 11r - 9r  - 6  + 14

In standard form, the polynomial is represented as follows;

M + T = -15r² + 2r + 8

learn more on polynomial here: https://brainly.com/question/12730389

#SPJ1

factor equation step by step.

Answers

Answer:

no solution

Step-by-step explanation:

use grouping method:

f(x) = x^2 - 4x + 2

factor it:

when we try to factor the above equation, we see that there is no ordered pair in which the numbers multiple to -4 and add to 2, so there is no solution

I’m so confused please help

Answers

Answer:

976cm squared

Step-by-step explanation:

that's all I could do .hope it helps. in case of any questions please ask.

NEED HELP ASAP

there should be a picture giving you info

Answers

Answer:

to find this radius you need to square the area and divide it by pi or 3.14 if your using Alex

If 3 times a certain number is increased by 4 the result is 28. What is the number?



If x is "the number," which of the following equations could be used to solve for the number?

3 x + 4 = 28
3 x = 4 + 28
3( x + 4) = 28

Answers

Answer:3x+4=28

Step-by-step explanation:

the second one would equal 3x=32

And the third one would equal 3x+12=28 which isn’t right either

At Hinsdale College, students are employed by the college library to re-shelve books.
The students claim that a 15-minute coffee break every hour would relive boredom and
increase the average number of books re-shelved per hour. To test the student's claim,
the following data were gathered on a random sample of seven student employees.
Over a period of one week, each student worked a normal schedule without coffee
breaks. Then, during the next week, the students were allowed coffee breaks.
Unknown to the students, a member of the library staff secretly recorded the average
number of books shelved per hour and computed the averages with and without the
coffee break system. The results are below. Test the claim that the students shelve
more books when they have a coffee break, use a 0.05 level of significance.
Student 1 2 3 4 5 6 7
Without
Coffee
breaks
75 63 82 53 79 96 73
With
Coffee
Breaks
90 72 95 50 85 110 89

Answers

The p value is less, therefore one should reject the null hypothesis.

How to illustrate the information?

From the information given, the test statistic will be:

t = (10 - 0)/(6.733/✓7)

t = 3.930.

The degree of freedom will be:

= 7 - 1 = 6

Using the table, the p value will be 0.0039.

Since the p value is less than 0.05 one should reject the null hypothesis.

Learn more about hypothesis on:

brainly.com/question/11555274

#SPJ1

Find the range of the function f(n) = n² - 2n for the domain (1, 3, 5).

Range:

Answers

Answer: {-1, 3, 15}

Step-by-step explanation:

[tex]f(1)=-1\\\\f(3)=3\\\\f(5)=15[/tex]

So, the range is {-1, 3, 15}.

What is addition and subtraction

Answers

[tex]\huge\text{Hey there!}[/tex]

[tex]\huge\textsf{What is addition \& subtraction?}[/tex]

[tex]\huge\textbf{What is addition?}[/tex]

[tex]\bullet\large\textbf{ Well, \underline{addition} simply means when you're ADDING}\\\large\textbf{2 or more numbers from each other to get the result of}\\\large\textbf{a number.}[/tex]

[tex]\huge\textbf{Random notes}[/tex]

[tex]\star\large\textbf{ Usually, you start working with addition when you're in}\\\large\textbf{pre-kindergarten (pre-k also known as head-start)}\\\\\star\large\textbf{ When, you're adding you are going upward.}\\\\\star\large\textbf{ Addition sometimes come after you learn how to count}\\\large\textbf{the basic numbers and so forth because adding never}\\\large\textbf{ends if you really think about.}[/tex]

[tex]\huge\textbf{Example:}[/tex]

[tex]\heartsuit\large\textbf{ 2 \boxed{\bf +} 5 = 7}\\\\\\\heartsuit\large\textbf{ You got the result of 7 because you started at 2 \& went}\\\larg\textbf{up 5 spaces to your right on your number and you landed}\\\large\textbf{on 7.}[/tex]

[tex]\huge\textbf{What is subtraction?}[/tex]

[tex]\bullet\large\textbf{ Well, subtraction simply means you're taking away. Often}\\\large\textbf{people call it take aways.}[/tex]

[tex]\huge\textbf{Random notes:}[/tex]

[tex]\star\large\textbf{ Usually, you start working with subtraction when you're}\\\large\textbf{in pre-kindergarten (pre-k also known as headstart)}\\\\\star\large\textbf{When you're subtracting you are doing downward.}\\\\\star\large\textbf{You focus more on subtraction when you're counting}\\\large\textbf{backwards.}[/tex]

[tex]\huge\textbf{Example:}[/tex]

[tex]\heartsuit\large\textbf{ 7 \boxed{\bf -} 5 = 2 }\\\\\\\heartsuit\large\textbf{ 7 \boxed{\bf -} 2 = 5}\\\\\\\heartsuit\large\textbf{ For equation \#1. you begin at 7 and go BACK 5 spaces}\\\large\textbf{and you should have landed on 2.}\\\\\\\heartsuit\large\textbf{ For equation \#2. you begin at 7 \& go BACK 2 spaces}\\\large\textbf{and you should have landed on 5.}[/tex]

[tex]\huge\textsf{KEEP IN MIND:}[/tex]

[tex]\circ\large\textsf{ Addition OPPOSITE is subtraction \& subtraction OPPOSITE is}\\\large\textsf{addition.}[/tex]

[tex]\huge\text{Good luck on your assignment \& enjoy your day!}[/tex]

~[tex]\frak{Amphitirite1040:)}[/tex]

Well add 2 numbers and get an answer and use the answer and minus one of the others numbers it would be the opposite

Graph the line that passes through the two points. Then find the slope of the line. (2,9), (4, 18)

Answers

The slope of the line is 4.5

How to determine the slope?

The points are given as:

(2,9) and (4, 18)

The slope (m) is calculated using:

m = (y2 - y1)/(x2 - x1)

This gives

m = (18 - 9)/(4 -2)

Evaluate

m = 4.5

Hence, the slope of the line is 4.5

Read more about slope at:

https://brainly.com/question/3493733

#SPJ1

3. Which graph represents the solution to 13k-21> 7?

Answers

Answer: Option (2)

Step-by-step explanation:

[tex]|3k-2| > 7[/tex] means [tex]3k-2 < -7[/tex] or [tex]3k-2 > 7[/tex].

Solving each of these cases gives [tex]k < -\frac{5}{3}[/tex] or [tex]k > 3[/tex].

So, the answer is Option (2).

put the events in order of what is most likely to happen, to least likely to happen. If you were to win a million dollars for winning one of these events, which would you pick to play? Use probability to explain your answers

Answers

Answer:

no man you should reconsider the question again

try typing again i guess its wrong

Step-by-step explanation:

no man you should reconsider the question again

try typing again i guess its wrong

winning a lottery would be correct one i guess

Perform the operation and simplify

Answers

Answer:

[tex]\frac{(x-2)(x+4)}{x+2}[/tex]

Step-by-step explanation:

[tex]\frac{(x+4)^2}{x+2} :\frac{(x+2)(x+4)}{(x+2)(x-2)} =\frac{(x+4)^2(x+2)(x-2)}{(x+2)(x+2)(x+4)}=\frac{(x-2)(x+4)}{x+2} .[/tex]

Solve the quadratic equation by using the quadratic formula. If the solutions are not real, enter NA.



2x2−7x+1=0


Enter the exact answers.



The field below accepts a list of numbers or formulas separated by semicolons (e.g. 2;4;6 or x+1;x−1 ). The order of the list does not matter.



Enclose numerators and denominators in parentheses. Remember that (a−b)/1+n is not the same as (a−b)/(1+n) .

Answers

By using the quadratic formula, the roots of the quadratic equation 2 · x² - 7 · x + 1 are represented by the irrational number x = (7 ± √41) / 4.

How to find the roots of quadratic equation by quadratic formula

For polynomials of the form a · x² + b · x + c = 0, the two roots can be found analytically by quadratic formula, whose expression is described below:

[tex]x = \frac{-b \pm \sqrt{b^{2}-4\cdot a\cdot c}}{2\cdot a}[/tex]    

If we know that a = 2, b = - 7 and c = 1, then the roots of the polynomial are:

x = 7/4 ± (1/4) · √41

x = (7 ± √41) / 4

By using the quadratic formula, the roots of the quadratic equation 2 · x² - 7 · x + 1 are represented by the irrational number x = (7 ± √41) / 4.

To learn more on quadratic equations: https://brainly.com/question/17177510

#SPJ1

MD writes an order for Lortab Elixir 5mg by mouth as needed for pain every 4 hours. Pharmacy dispenses you with 7.5mg/15ml. How many tablespoons will you administer per dose?

Answers

Following the prescription, You will need to administer a 0.7 tablespoon per dose.

Determining the number of tablespoons by using equivalent expressions?

We can determine the number of tablespoons that are administered by using equivalent expressions.

From the standard conversion rate, we know that:

1 ml = 0.067628 tbsp15 ml = 1.01442 tbsp

Using equivalent expressions:

7.5 mg/1.01442 tbsp = 5 mg/x tbsp

x = (1.01442 × 5)/7.5 tbsp

x = 0.67 tbsp

x = 0.7 tbsp per dose.

Learn more about equivalent expressions here:

https://brainly.com/question/24734894

#SPJ1

A company manufactures both mountain bikes and trail bikes. The cost of materials for a mountain bike is $40, and the cost of materials for a trail bike is $40. The cost of labor to manufacture a mountain bike is $90, and the cost of labor to manufacture a trail bike is $30. During a week in which the company has budgeted $1,600 for materials and $2,700 for labor, how many mountain bikes does the company plan to manufacture?

Answers

An equation is formed when two equal expressions. The company is planning to produce 25 mountain bikes and 15 trail bikes.

What is an equation?

An equation is formed when two equal expressions are equated together with the help of an equal sign '='.

Let the number of mountain bikes produced by the company be x, while the number of trail bikes produced by the company is y.

The cost of materials for a mountain bike is $40, and the cost of materials for a trail bike is $40. Also, the budget of the company is $1,600. Therefore,

1600 = 40x + 40y

x + y = 40

y = 40-x

The cost of labour to manufacture a mountain bike is $90, and the cost of labour to manufacture a trail bike is $30. Also, the budget for labour is $2,700.

2700 = 90x + 30y

2700 = 90x + 30(40-x)

2700 = 90x + 1200 - 30x

1500 = 60x

x = 25

Substitute the value of x,

y = 40 - x

y = 15

Hence, the company is planning to produce 25 mountain bikes and 15 trail bikes.


Learn more about Equation:

https://brainly.com/question/2263981

#SPJ1

f(x)=3√x + 7. Find the inverse of f(x).

Answers

f(x)=(x^2-14x+49)/9


step 1- switch x and y
step 2- isolate y, and get to (x-7)/3=sqrty
step 3- square each side to get rid of the square root and isolate y
step 4- f(x)=(x^2-14x+49)/9

Answer:

[tex]f^{-1}(x)=\left(\dfrac{x-7}{3}\right)^2[/tex]

Step-by-step explanation:

Given function:

[tex]f(x)=3\sqrt{x}+7[/tex]

To find the inverse of the function

Swap f(x) for y:

[tex]\implies y=3 \sqrt{x}+7[/tex]

Subtract 7 from both sides:

[tex]\implies y-7=3 \sqrt{x}+7-7[/tex]

[tex]\implies y-7=3\sqrt{x}[/tex]

Divide both sides by 3:

[tex]\implies \dfrac{3\sqrt{x}}{3}=\dfrac{y-7}{3}[/tex]

[tex]\implies \sqrt{x}=\dfrac{y-7}{3}[/tex]

Square both sides:

[tex]\implies \left(\sqrt{x}\right)^2=\left(\dfrac{y-7}{3}\right)^2[/tex]

[tex]\implies x=\left(\dfrac{y-7}{3}\right)^2[/tex]

Swap the x for [tex]f^{-1}(x)[/tex] and y for x:

[tex]\implies f^{-1}(x)=\left(\dfrac{x-7}{3}\right)^2[/tex]

This is a cross-sectional view of candy bar ABC. A candy company wants to create a cylindrical container for candy bar ABC so that it is circumscribed about the
candy bar. If AD = 2.5 cm, what is the smallest diameter of wrapper that will fit the candy bar?

Answers

TheThe smallest diameter of wrapper that will fit the candy bar is 5cm.

Smallest diameter

Given:

AD = DC

AC = AD + DC

Segment AD = 2.5cm

Hence:

AC = AD + AD (AD = DC)

AC = 2AD

Where:

AD=2.5

So,

AC = 2(2.5cm)

AC = 5cm

Therefore the smallest diameter of wrapper that will fit the candy bar is 5cm.

Learn more about smallest diameter here: https://brainly.com/question/12944899

#SPJ1

Answer: 5cm.

Step-by-step explanation:

Factor by Grouping.
25x^2-20x-12

Answers

Answer:

ac = -300-20 = -30 +10(5x +2)(5x +(-6))

Step-by-step explanation:

The sequence of required answers provides a step-by-step solution to the problem.

Factors of ac

In the given trinomial, we identify the coefficients as ...

  ax² +bx +c = 25x² -20x -12   ⇒   a=25; b=-20; c=-12

Then the ac product is ...

  ac = (25)(-12) = -300

The factor pairs that make this product can be listed as ...

  -300 = (-300)(1) = (-150)(2) = (-100)(3) = (-75)(4) = (-60)(5) = (-50)(6) = (-30)(10) = (-25)(12) = (-20)(15) . . . . (only pairs with a negative sum are listed)

The sums of these pairs are -299, -148, -97, -71, -55, -44, -20, -13, -5.

The factors of the ac product that add to -20 are -30 and 10.

Factor by grouping

These factors are used to rewrite the -20x term as a sum.

  25x² -20x -12 = 25x² -30x +10x -12

Grouping these terms in pairs, we can identify common factors:

  = (25x² -30x) +(10x -12)

  = 5x(5x -6) +2(5x -6)

  = (5x +2)(5x +(-6))

What is the following quotient?
2
√13+√11
O√13-2√11
√13+√11
6
O
113 + √11
12
O
● √13 - √11

Answers

[tex]\frac{2}{\sqrt{13}+\sqrt{11}} \times \frac{\sqrt{13}-\sqrt{11}}{\sqrt{13}-\sqrt{11}}\\\\=\frac{2\sqrt{13}-2\sqrt{11}}{13-11}\\\\=\boxed{\sqrt{13}-\sqrt{11}}[/tex]

The solution is: our final quotient is:  (√13 - √11).

What is  multiplication?

In mathematics, multiplication is a method of finding the product of two or more numbers. It is one of the basic arithmetic operations, that we use in everyday life.

here, we have,

Explanation

Since we have radicals in the denominator, we need to rationalize our expression.

To do it we just need to multiply both numerator and denominator by the conjugate of the denominator.

Remember that the conjugate of a binomial is the same binomial with the sing in between changed.

given that,

2/ (√13 + √11)

Now, we can simplify the denominator:

multiplying by  (√13 - √11)

we get,

2 *  (√13 - √11) / (√13 + √11)*  (√13 - √11)

The only radical that we can simplify in the numerator is , which simplifies to:

2 (√13 - √11) / 13 - 11

=  2(√13 - √11) /2

= (√13 - √11)

So, our final quotient is:  (√13 - √11)

To learn more on multiplication click:

brainly.com/question/5992872

#SPJ5

What is the value of g(f(x)) when x=-2 if f(x)=x3

Answers

Answer:

The answer would be A: -15

5. Lucinda is a babysitter. Her hourly rate is $5.00. fin one week, she baby sat 13 hours and
received tips of $14.50. How much did she earn?

Answers

5.00 x 13 hours = 65
$65 + $14.50 = $79.50

The Robinsons family drove for 4000 miles. This was the
distance the Jones family drove. How many miles did the Jones
family drive?

Answers

Answer:1333.3 Contiuous

Step-by-step explanation:

 .

4000/3,=1333.3x1,=1333.3

 I'm not sure

-3x + 2y = 9
x - 3y = 4

Answers

Answer:

(-5, -3)

Step-by-step explanation:

Isolate x:

x - 3y = 4

x = 3y + 4

Substitute to get y:

-3x + 2y = 9

-3(3y + 4) + 2y = 9

-9y - 12 + 2y = 9

-7y = 21

y = -3

Substitute to get x:

x = 3y + 4

= 3(-3) + 4

-9 + 4

= -5

Other Questions
A beneficial rule to follow is to set the firm's capital structure so that Blank______. Multiple choice question. the firm's value is minimized the firm's bondholders are satisfied the firm's value is maximized dividends are maximized When they laughed at my feet, they did not realize that I had put on two different shoes because Id had to dress in the dark that morning. They did not realize that I had dressed in the dark so I wouldnt wake my father, who works nights to put food on our table. They did not realize that I had dressed in the dark to not disturb my mother, who had been up all night with the newborn twins. They just laughed.How does the passage develop the conflict? How does the use of "peevish schoolboy help the reader understand Cassiuss feeling toward Octavius?It reveals a deeper level of contempt when saying Octavius is but an annoying child.It emphasizes his belief that Octavius is a respected politician.It suggests sympathy because he thinks that Octavius has been deceived.It suggests a strong fear of Octavius deadly skills as a warrior. 6) The circumference of a circle isapproximately 37.68 centimeters. What is the radius of the circle to the nearest centimeter?A. 6 cmC. 12 cmB. 8 cmD. 16 cm what was the most defining characteristic of the Texas Right in the twentieth century? Explain. Moss County Bank agrees to lend the Sadowski Brick Company $500,000 on January 1. Sadowski Brick Company signs a $500,000, 6%, 9-month note. The entry made by Sadowski Brick Company on January 1 to record the proceeds and issuance of the note is A wire that is 22 feet long connects the top of a pole to the ground. The wire is attached to the ground at a point that is 10 feet from the base of the pole. What is the length of the pole, round o the nearest tenth.a. 12.00 ftb. 19.60 ftc. 24.17 ftd. 38.40 ftNEED HELP ASAP Molecular and empirical formulas as a superhero it is possible that some people hate you or spread rumors about you how do you deal with those people? Simplify the expression (startfraction 1 over 4 a b endfraction) superscript negative 2. assume a not-equals 0, b not-equals 0. negative startfraction 1 over 16 a squared b squared endfraction startfraction a squared b squared over 4 endfraction negative 16 a squared b squared 16 a squared b squared what is the next # in this sequence 3, 5, 12, 55, 648 Which of the following WWII advancementsallowed planes to be tracked during air raids?A. OSRDC. SONARB. RADARD. USAF A wood pole is 12 metres long and is cut into 5 equal sections. If each cut wastes 10 cm of pole, what length would each section be? * Copy the figure below onto a separate sheet of paper. Find the image of the figure under reflections in line m and then line t. In the box below, descibe the new location of point F in relation to lines m and t. I am not able to solve this question (Contacts) An offer need not be reasonable to be valid Which of the following is correct equation for the trend line in the scatter plot? A. Y= 2/5 x -2 B. Y= 5x-1 C. Y= 5x +5 D. Y= -x+5 Which graph represents y > 2x^2 8x + 4? The elderly tend to become less physically active because of social isolation. Group of answer choices True False This table shows equivalent ratios.A 2-column table has 4 rows. Column 1 is labeled A with entries 12, 24, 32, 48. Column 2 is labeled B with entries 2, 4, 6, 8.Which ratio is not equivalent to the other ratios shown in this table?12 to 224 to 432 to 648 to 8